Cálculo del valor esperado de vacío en la teoría de la perturbación quiral

Desde un punto de vista, tengo que admitir desde el principio que esta pregunta puede ser un problema de cálculo trivial, pero la verdad es que estoy bastante atascado. Estoy estudiando la teoría de la perturbación quiral con el objetivo de construir la x PAG T Lagrangiano. Por esta razón, examino primero la apariencia del condensado de quarks.

Más detalladamente, tenemos la densidad pseudoescalar PAG i = q ¯ γ 5 τ i q y estamos trabajando solo con los quarks arriba y abajo, por lo que la simetría completa del QCD Lagrangiano es S tu ( 2 ) × S tu ( 2 ) .

Mi problema es cómo debo calcular el VEV de P. Es decir, el hecho de que las matrices de Pauli τ i son 2 × 2 dimensional y el γ 5 es 4 × 4 . Trabajo sobre la base de Dirac para la γ 5 de modo que es cero en la diagonal y I 2 × 2 en las otras entradas. No sé si de alguna manera debo usar el hecho de que la u y la d son espinores de Dirac o, como encuentro en la página 81 de https://arxiv.org/abs/hep-ph/0210398 , use una relación similar a la relación 4.17 - que de hecho no entiendo.


EDITAR

Para ponerlo en una imagen más matemática, lo que deseamos calcular es algo de la forma

q ¯ γ 5 τ 3 q
donde esta q
q = ( tu d ) = ( tu 1 tu 2 tu 3 tu 4 d 1 d 2 d 3 d 4 ) ,
es decir, tanto u como d son espinores de Dirac. Pero tengo en cuenta que la matriz gamma es 4 × 4 y la matriz de Pauli es 2 × 2 , que me parece bien porque actúa solo en los componentes isospin u y d.

Entonces, ¿cómo debo realizar la acción en el γ 5 matriz en el q vector, que es de hecho un espinor de Dirac de dos componentes?

Gracias.

Ummm... las P son campos de interpolación QCD para los mesones pseudoescalares, los piones, mientras que las S para los mesones escalares... la sigma... así que (4.17-18) conecte las P con las S a través de la acción de una carga axial, que no aniquila el vacío.... Es el rhs de (4.17-18) el que tiene un vev no trivial, el condensado quiral, y no lo calculas, a menos que hagas la teoría de calibre reticular. La tabla de la página 81 te dice qué es, y lo correlacionas con las propiedades de los pseudoescalares...

Respuestas (1)

Como indiqué, no deseo hacer muecas en su artículo de revisión, pero el texto de TP Chang & LF Li , Ch 5.4, 5.5, es menos hostil.

En cualquier caso, parece perdido, y un mapa de ruta de la disposición del terreno está en orden , así que seré esquemático, ya que todas las fórmulas correctas están ahí, pero su conectividad parece perdida... Todos los índices de matriz γ en el producto directo γ τ a están saturados por espinores, por lo que solo se trata de combinaciones escalares y pseudoescalares con sabor S a , P a , y sin sabor tales como: S, P . (Entonces, para responder a su pregunta secundaria de edición, el γ 5 la acción es sabor diagonal: conecta u s con u s y d s con d s.)

El punto es no perturbativo QCD produce un condensado escalar, es decir, solo los Ss toman un vev y nunca los Ps , la última línea de la tabla 4.1, p. 81, entonces S = q ¯ q 0 , lo que la gente normal llama σ .

Así que, para empezar, te estás preguntando cómo calcular un 0, que en realidad es un hecho. QCD, como teoría vectorial, no puede romper la paridad. Para empezar, tampoco puede romper el isospín (que es roto explícitamente por las masas de los quarks pequeños), así que relaciona las componentes vectoriales de S a = 0 , para producir restricciones adicionales sobre bilineales, como lo hace en (4.15), para isospin solo tu ¯ tu = d ¯ d .

No está calculando el vev de piones, sino, efectivamente, de bilineales de tales objetos (cada uno de ellos un quark bilineal). Sin sumatoria sobre índices de isospín repetidos, (4.17,8), d a PAG a 0 , produce la versión no integrada que no desaparece de (4.19). La carga axial no puede aniquilar el vacío, ni tampoco la densidad pseudoescalar P a .

Ahora, los bilineales pseudoescalares PAG a son los campos de interpolación QCD de los 3 piones, π , los campos de Goldstone de esta ruptura de simetría quiral espontánea/dinámica, y definitivamente no tienen vevs que no desaparecen; recuerde el modelo quiral σ en las secciones anteriores, a pesar de que no aniquiles el vacío, por arriba.

Pero conocen el vacío... En lugar de matarlo, excluido anteriormente, se deslizan dentro y fuera de él, a través de la ecuación más importante del artículo, y de cualquier artículo sobre el tema, (4.19), PCAC, la madre de todos los teoremas de piones blandos en el álgebra actual ,

0 | A m a ( 0 ) | π b ( pag ) = i pag m F d a b .

El campo axial en el medio es la corriente de carga quiral, por lo que, en la transformada de Fourier, su componente 0 integrado en el espacio daría 0 actuando a la izquierda, si el vacío fuera quiralmente invariante, ¡pero no lo es! (En realidad, la carga quiral tiene problemas de infrarrojos en su definición, según el teorema de Fabry-Picasso , pero no entremos en líos...) Lo que hace es generar un pión en esta realización no lineal, que aniquila al pión sobre el que actúa, A la derecha.

Entonces, tomando la divergencia de esta corriente en el espacio de Fourier, obtenemos metro π 2 en el lado derecho, y eso debería desaparecer en un mundo ideal (CAC) en el que m A m a = 0 . Pero este es el sucio mundo real en el que la simetría quiral es aproximada hasta las masas de los quarks, por lo que la masa del pión puede sobrevivir hasta ser distinta de cero (P de "parcialmente") siempre que haya pequeñas masas de quarks "semilla" para evitar que se produzca. siendo un goldston, como lo requiere el teorema de Goldstone: los piones son pseudo-goldstons.

Su vev se está desvaneciendo en un solo vacío, |0>, pero la carga quiral se atasca en el vacío y crea estados de frecuencia casi (pseudo) cero de ellos.

Gracias profesor. Me preguntaba si también podría recibir un comentario tuyo sobre esto: physics.stackexchange.com/questions/355827/… . Pero, de todos modos, su consideración ha sido de gran ayuda.